Ableitung einer Antikommutierungsbeziehung zwischen Erzeugungs-/Vernichtungsoperatoren für Dirac-Fermionen

Ausgehend von Dirac-Feldern:

Ψ ( X ) = 1 ( 2 π ) 3 / 2 D 3 k 2 ω k R [ C R ( k ) u R ( k ) e ich k X + D R ( k ) v R ( k ) e ich k X ] k 0 = ω k

Ψ ( X ) = 1 ( 2 π ) 3 / 2 D 3 k 2 ω k R [ D R ( k ) v R ( k ) e ich k X + C R ( k ) u R ( k ) e ich k X ] k 0 = ω k

Wo ω k = k 2 + M 2 .

Die kanonische Quatisierungsbedingung lautet:

{ { Ψ a ( X ) , Ψ β ( j ) } T = δ ( 3 ) ( X j )     δ a β { Ψ a ( X ) , Ψ β ( j ) } T = 0 { Ψ a ( X ) , Ψ β ( j ) } T = 0

Um die Quantisierungsbedingung für die Erzeugungs-/Vernichtungsoperatoren abzuleiten, muss ich umschreiben C , C , D , D bezüglich Ψ Und Ψ .

Beispielsweise um die kanonische Quantisierungsbedingung zwischen abzuleiten C , C Ich kann sie umschreiben als:

C R ( k ) = 1 2 π 3 D 3 X 2 ω k u R ( k ) Ψ ( X ) e ich k X

C S ( P ) = 1 2 π 3 D 3 j 2 ω P Ψ ( j ) u S ( P ) e ich P j

und dann explizit den Antikommutator berechnen:

{ C R ( k ) , C S ( P ) } T = 1 ( 2 π ) 3 D 3 X D 3 j 2 ω k 2 ω P [ u R ( k ) Ψ ( X ) Ψ ( j ) u S ( P ) + Ψ ( j ) u S ( P ) u R ( k ) Ψ ( X ) ] e ich ( k X P j ) = 1 ( 2 π ) 3 D 3 X D 3 j 2 ω k 2 ω P [ u R ( k ) { Ψ ( X ) , Ψ ( j ) } u S ( P ) ] e ich ( k X P j ) =

Aber hier vermisse ich etwas: Ich verstehe nicht, warum ich tauschen kann u S ( P ) Und u R ( k ) im zweiten Term, um den Antikommutator dazwischen wiederzugewinnen Ψ Und Ψ .

In allen heiligen Lehrbüchern ist es umgekehrt. Sie postulieren die Antikommutierungsrelationen für C R Und D R und dann sind Sie nur noch einen Schritt davon entfernt, die Antikommutierungsbeziehungen für die Felder abzuleiten.

Antworten (2)

Sie sollten in Kontraktion der Spinor-Indizes vorgehen und sich daran erinnern, dass zB ein Paar u ( k , R ) Ψ ( X ) u a ( k , R ) Ψ a ( X ) , mit u a ( k , R ) Kennzeichnung der a th Bestandteil des Dirac-Spinors u ( k , R ) . Deshalb,

{ C ( k , R ) , C ( P , S ) } D 3 X D 3 j ( u a ( k , R ) Ψ a ( X ) Ψ β ( j ) u β ( P , S ) + Ψ β ( j ) u β ( P , S ) u a ( k , R ) Ψ a ( X ) ) = D 3 X D 3 j ( u a ( k , R ) { Ψ a ( X ) , Ψ β ( j ) } u β ( P , S ) ) = =

Beenden Sie die Übung mit Ihren kanonischen Quantisierungsfeld-Antikommutatoren zusammen mit Orthogonalitäts-/Vollständigkeitsbeziehungen zwischen den u 'S.

Hier Ψ ( X ) ist eine Spaltenmatrix mit 4 Komponenten und Ψ ( X ) ist eine Zeilenmatrix mit 4 Zeilenelementen (natürlich sind diese Elemente Funktionen von X ).

Und wenn Sie die Anti-Kommutation nehmen, wählen Sie eine Komponente (oder ein Element) Ψ a ( X ) ab (4 × 1) Spaltenmatrix Ψ ( X ) und in ähnlicher Weise sollten Sie eine Komponente auswählen Ψ β ( X ) ab (1 × 4) Zeilenmatrix Ψ ( X ) .

Ψ a ( X ) = 1 ( 2 π ) 3 / 2 D 3 k 2 ω k R = 1 , 2 [ C R ( k ) u R , a ( k ) e ich k X + D R ( k ) v R , a ( k ) e ich k X ] k 0 = ω k Ψ β ( X ) = 1 ( 2 π ) 3 / 2 D 3 k 2 ω k R = 1 , 2 [ D R ( k ) v R , β ( k ) e ich k X + C R ( k ) u R , β ( k ) e ich k X ] k 0 = ω k
So, jetzt können Sie dort sehen u Und v hat zwei Indizes r und a (oder β ), Hier R kann dabei die Werte 1 und 2 annehmen a (oder β ) kann die Werte 1,2,3,4 annehmen.

Oder einfach u R , a (oder v R , a ) ist der a -te Komponente (bzw a -tes Matrixelement) der (4 × 1) Spaltenmatrix u R (oder v R ).

Und ähnlich u R , a (oder v R , a ) ist der a -te Komponente (bzw a -tes Matrixelement) der (1 × 4) Zeilenmatrix u R (oder v R ).

Daher u R , a , v R , a , u R , a , v R , a all dies sind nur Zahlen oder die Matrixelemente, nicht die Matrizen.

Sie fahren also wie gewohnt fort und tauschen einfach aus u S , a ( P ) Und u R , a ( k ) (in Ihrer Notation), da es sich nur um die Zahlen oder die Komponenten (oder Elemente) der entsprechenden Matrizen handelt.